관리 메뉴


수악중독

2008 AMC12 A #5 본문

AMC12/2008

2008 AMC12 A #5

수악중독 2014. 12. 19. 17:13

Suppose that \[\dfrac{2x}{3}-\dfrac{x}{6}\] is an integer. Which of the following statements must be true about \(x\)?


(A) It is negative.

(B) It is even, but not necessarily a multiple of \(3\).

(C) It is a multiple of \(3\), but not necessarily even.

(D) It is a multiple of \(6\), but not necessarily a multiple of \(12\).

(E) It is a multiple of \(12\).



Comments